Nursing Fundamentals - EXAM 2

Pataasin ang iyong marka sa homework at exams ngayon gamit ang Quizwiz!

The client had diminished wheezing in both lungs after receiving emergency treatment for an acute asthma attack. When utilizing focus charting, in which section should the nurse document this information? 1. Data (D) 2. Action (A) 3. Response (R) 4. Planning (P)

Answer: 3

Describing skin lesions

1. Distribution 2. Location 3. Pattern 4. Exudate (what kind...color, smell, blood?) 5. Appearance (color, size, shape, texture, firmness)

A client comes to the walk-in clinic with reports of abdominal pain and diarrhea. While taking the client's vital signs, the nurse is implementing which phase of the nursing process? A. Assessment B. Diagnosis C. Planning D. Implementation

A. Assessment Rationale: The first step in the nursing process is assessment, the process of collecting data. All subsequent phases of the nursing process (options 2, 3, and 4) rely on accurate and complete data.

At what age should women start screening for cervical and uterine cancer?

Ages 21-29, screening with Pap smear every 3 years. Ages 30-65, screening every 5 years with both HPV and PAP 65 +, or if you have had 3 or more consecutive NEG results every 10 years.

A major characteristic of the nursing process is which of the following? 1. A focus on client needs 2. Its static nature 3. An emphasis on physiology and illness 4. Its exclusive use by and with nurses

Answer: 1. Rationale: The nursing process focuses on client needs. It is dynamic rather than static

During an assessment, a client who is not very talkative appears pale, diaphoretic, and restless in the bed and says, "Leave me alone." Which subjective data should the nurse document? 1. Restlessness 2. "Leave me alone" 3. Not talkative 4. Pale and diaphoretic

Answer: 2 Explanation: Subjective data can be described or verified only by that person and are apparent only to the person affected. Subjective data include the client's sensations, feelings, beliefs, attitudes, and perceptions of personal health status and life situations.

The nurse makes the following entry on the client's care plan: "Goal not met. Client refuses to ambulate, stating, 'I am too afraid I will fall.' " The nurse should take which of the following actions? A. Notify the physician B. Reassign the client to another nurse C. Reexamine the nursing orders D. Write a new nursing diagnosis

B. Reexamine the nursing orders Rationale: The plan needs to be reassessed whenever goals are not met. Nursing interventions should be examined to ensure the best interventions were selected to assist the client achieve the goal. The goal may be appropriate, but the client may need more time to achieve the desired outcome. The manner in which the nursing interventions were implemented may have interfered with achieving the outcome.

Cranial Nerve VII

Facial - controls most facial expressions & secretion of tears & saliva & taste

Skin turgor

Measure of hydration, which tests how quickly the skin returns to its normal position after being pinched

Cranial Nerve VIII

Vestibulocochlear (hearing and balance)

Anisocoria

a condition in which the pupils are unequal in size

Percussion

a diagnostic procedure designed to determine the density of a body part by the sound produced by tapping the surface with the fingers

Tuning fork

a two-pronged, fork-like instrument that vibrates when struck; used to test hearing, especially bone conduction

Ophthalmoscope

an instrument used to examine the interior of the eye

Percussion hammer

an instrument with a rubber head, used for testing reflexes

PERRLA

pupils equal, round, reactive to light and accommodation

Nodule

solid, round or oval elevated lesion 1 cm or more in diameter

A client is coming into the clinic for the first time. In order for the nurse to allow the client the most comfort during the interview, what should the nurse do? 1. Sit next to the client, a few feet apart. 2. Sit behind a desk. 3. Stand at the side of the client's chair. 4. Stand at the counter to take notes during the interview.

Answer: 1 Explanation: A seating arrangement in which the client and nurse are seated in chairs, a few feet apart, at right angles to each other and with no table between, creates a less formal atmosphere, with the nurse and client feeling on equal terms. This would allow for more comfort and relaxation during the interview phase.

While conducting a dressing change, the nurse notes a new area of skin breakdown that was caused from the tape used to secure the dressing. In which phase of the nursing process is the nurse working? 1. Assessment 2. Diagnosis 3. Implementation 4. Evaluation

Answer: 1 Explanation: Assessment is the collection, organization, validation, and documentation of data. Assessment is carried throughout the nursing process, as in this case. Even though performing the dressing change is implementation, noticing the new skin breakdown is assessment.

Which of the following would indicate a significant cue when comparing data to standards? Select all that apply. 1. The client has moved partway toward a set goal (e.g., weight loss). 2. The client's vision is within normal range only when wearing glasses. 3. A child is able to control bladder and bowels at age 18 months. 4. A recently widowed woman states she is "unable to cry." 5. A 16-year-old high school student reports spending 6 hours doing homework 5 nights per week.

Answer: 1, 4, and 5. Rationale: A client's movement toward a goal (option 1) or whose behavior is inconsistent with population norms (options 4 and 5) represents a cue that further analysis toward creating a nursing diagnosis is required. Corrected vision (option 2) and bladder and bowel control at age 18 months (option 3) are consistent with population norms

A collaborative (multidisciplinary) problem is indicated instead of a nursing or medical diagnosis 1. If both medical and nursing interventions are required to treat the problem. 2. When independent nursing actions can be utilized to treat the problem. 3. In cases where nursing interventions are the primary actions required to treat the problem. 4. When no medical diagnosis (disease) can be determined.

Answer: 1. Rationale: A collaborative (multidisciplinary) problem is indicated when both medical and nursing interventions are needed to prevent or treat the problem. If nursing care alone (whether that care involves independent or dependent nursing actions) can treat the problem, a nursing diagnosis is indicated

. In the case in which a client is vulnerable to developing a health problem, the nurse chooses which type of nursing diagnosis status? 1. A risk nursing diagnosis 2. A syndrome nursing diagnosis 3. A health promotion nursing diagnosis 4. An actual nursing diagnosis

Answer: 1. Rationale: A risk nursing diagnosis is appropriate when the evidence for the problem indicates that a condition exists that makes the client vulnerable to a problem

Which of the following is the purpose of assessing? 1. Establish a database of client responses to his or her health status. 2. Identify client strengths and problems. 3. Develop an individualized plan of care. 4. Implement care, prevent illness, and promote wellness.

Answer: 1. Rationale: Assessing provides a database of the client's physiologic and psychosocial responses to his or her health status.

1. Which behavior is most representative of the nursing diagnosis phase of the nursing process? 1. Identifying major problems or needs 2. Organizing data in the client's family history 3. Establishing short-term and long-term goals 4. Administering an antibiotic

Answer: 1. Rationale: Identifying problems and needs is part of a nursing diagnosis. For example, a client with difficulty breathing would have constricted airways as manifested by shortness of breath (dyspnea) as a nursing diagnosis.

Which nursing diagnosis contains the proper components? 1. Potential for impairment in caregiver role related to unpredictable illness course 2. Potential for falls related to tendency to collapse when having difficulty breathing 3. Altered communication related to stroke 4. Altered sleep secondary to fatigue and a noisy environment

Answer: 1. Rationale: States the relationship between the stem (impairment in caregiver role) and the cause of the problem.

. Which of the following represent effective planning of the interview setting? Select all that apply. 1. Keep the lighting dimmed so as not to stress the client's eyes. 2. Ensure that no one can overhear the interview conversation. 3. Stand near the client's head while he or she is in the bed or chair. 4. Keep approximately 3 feet from the client during the interview. 5. Use a standard form to be sure all relevant data are covered in the interview.

Answer: 2, 4, and 5. Rationale: The nurse plans the interview so that privacy is observed. A comfortable distance between nurse and client to respect the client's personal space is about 3 feet. Using a standard form will help ensure the nurse doesn't omit gathering any vital information. Lighting should be at a normal level—neither bright nor dim

. In the diagnostic statement "increased fluid volume related to decreased venous return as manifested by lower extremity edema (swelling)," the etiology of the problem is which of the following? 1. Increased fluid volume 2. Decreased venous return 3. Edema 4. Unknown

Answer: 2. Rationale: Because the venous return is impaired, fluid is static, resulting in swelling. Therefore, decreased venous return is the cause (etiology) of the problem. Increased fluid volume is the nursing diagnosis, and edema of the lower extremity is the sign, symptom, or critical attribute.

The nurse is conducting the diagnosing phase (nursing diagnosis) of the nursing process for a client with a seizure disorder. Which step exists between data analysis and formulating the diagnostic statement? 1. Assess the client's needs. 2. Delineate the client's problems and strengths. 3. Determine which interventions are most likely to succeed. 4. Estimate the cost of several different approaches.

Answer: 2. Rationale: In diagnosing, data from assessment

Which element is best categorized as secondary subjective data? 1. The nurse measures a weight loss of 10 pounds since the last clinic visit. 2. Spouse states the client has lost all appetite. 3. The nurse palpates edema in lower extremities. 4. Client states severe pain when walking up stairs

Answer: 2. Rationale: Primary data come from the client

The nurse provides a back rub to a client after administering a pain medication with the hope that these two actions will help decrease the client's pain. Which phase of the nursing process is this nurse implementing? 1. Assessment 2. Diagnosis 3. Implementation 4. Evaluation

Answer: 3 Explanation: Implementation is that part of the nursing process in which the nurse applies knowledge to perform interventions.

Family of a client demonstrating confusion state that this is not the client's usual behavior. How should the nurse document this data? 1. Inference 2. Subjective data 3. Objective data 4. Secondary subjective data

Answer: 3 Explanation: Information supplied by family members, significant others, or other health professionals are considered subjective if it is not based on fact. Because this information is factual, in that the spouse is able to provide the nurse with information about the client's routine behavior and patterns, this is objective data.

The nurse is admitting an infant to the care area. The parents and grandmother are present. What should the nurse use as the best source of data for this client? 1. Medical record from the child's birth 2. Grandmother 3. Parents 4. Admitting physician

Answer: 3 Explanation: The best source of data is usually the client, unless the client is too ill, young, or confused to communicate clearly. The parents would be able to provide the nurse with the most accurate, current information regarding the baby (diet, schedule, symptoms, etc.).

The nurse has worked in the trauma critical care area for several years. Which noise may become indiscriminate for this particular nurse? 1. A client with audible breathing 2. Moaning of a client in pain 3. Whirring of ventilators 4. Coworkers discussing their clients' conditions

Answer: 3 Explanation: The noises of machines and other equipment noises-except alarms-would be easy to ignore, as these are the usual, normal sounds of the unit.

The student is learning the steps of the nursing process. What is the first thing that the student should realize about the purpose of this process? 1. Deliver care to a client in an organized way. 2. Implement a plan that is close to the medical model. 3. Identify client needs and deliver care to meet those needs. 4. Make sure that standardized care is available to clients.

Answer: 3 Explanation: The purpose of the nursing process is to identify a client's health status and actual or potential healthcare problems or needs, to establish plans to meet the identified needs, and to deliver specific nursing interventions to meet those needs. Page Ref: 164

A client has been using the call light routinely throughout the evening. Upon entering the room, the nurse observes the following details. Organize them according to priority sequencing (1 is first priority; 5 is least priority). 1. The family is at the bedside. 2. The IV pump is running on battery. 3. The ECG monitor shows tachycardia. 4. The client reports being restless. 5. O2 tubing is not attached to wall regulator.

Answer: 3, 4, 5, 2, 1 Explanation: 1. Has no apparent bearing on client's symptoms 2. Indicates an issue worth observing 3. Indicates an objective cardiac symptom 4. Indicates a subjective symptom 5. Indicates a possible cause of the client's symptoms

Which statement is true regarding the state of the science in regard to nursing diagnosis? 1. The original taxonomy has proven to be adequate in scope. 2. The organizing framework of the taxonomy is based on the work of Florence Nightingale. 3. More research is needed to validate and refine the diagnostic labels. 4. New diagnostic labels are approved by means of a vote of registered nurses.

Answer: 3. Rationale: Diagnostic labels are continuously reviewed and revised as indicated by research—much more of which is needed. The original taxonomy has been replaced by Taxonomy II and is no longer based on a nurse theorist (options 1 and 2). New diagnoses are approved by NANDA International's Diagnostic Review Committee, not by a vote of nurses (option 4)

. Which behavior would indicate that the nurse was utilizing the assessment phase of the nursing process to provide nursing care? 1. Proposes hypotheses. 2. Generates desired outcomes. 3. Reviews results of laboratory tests. 4. Documents care.

Answer: 3. Rationale: During assessment, data are collected, organized, validated, and documented. Hypotheses are generated during diagnosing; outcomes are set during planning; and documentation occurs throughout the nursing process.

The nurse wishes to determine the client's feelings about a recent diagnosis. Which interview question is most likely to elicit this information? 1. "What did the doctor tell you about your diagnosis?" 2. "Are you worried about how the diagnosis will affect you in the future?" 3. "Tell me about your reactions to the diagnosis." 4. "How is your family responding to the diagnosis?"

Answer: 3. Rationale: Eliciting feelings requires an open-ended question that does more than seek factual information

In the validating activity of the assessing phase of the nursing process, the nurse performs which of the following? 1. Collects subjective data. 2. Applies a framework to the collected data. 3. Confirms data are complete and accurate. 4. Records data in the client record.

Answer: 3. Rationale: In validating, the nurse confirms that data are complete and accurate. Subjective data are collected in the collecting activity

A new client has been admitted to the care area. How soon should the nurse plan to complete a physical assessment on this patient? 1. 1 hour 2. 12 hours 3. 48 hours 4. 24 hours

Answer: 4 Explanation: The Joint Commission requires that each client have an initial assessment consisting of a history and physical performed and documented within 24 hours of admission as an inpatient.

The use of a conceptual or theoretical framework for collecting and organizing assessment data ensures which of the following? 1. Correlation of the data with other members of the healthcare team 2. Demonstration of cost-effective care 3. Utilization of creativity and intuition in creating a plan of care 4. Collection of all necessary information for a thorough appraisal

Answer: 4. Rationale: Frameworks help the nurse be systematic in data collection. Other members of the healthcare team may use very different conceptual organizing frameworks so data may not correlate

Which statement would be true regarding use of the observing method of data collection? 1. When observing, the nurse uses only the visual sense. 2. Observing is done only when no other nursing interventions are being performed at the same time. 3. Data should be gathered as they occur, rather than in any particular order. 4. Observed data should be interpreted in relation to other sources of collected data

Answer: 4. Rationale: Interpreting collected data is necessary to help validate their accuracy. Observing includes the senses of smell, hearing, and touch in addition to vision

One of the primary advantages of using a three-part diagnostic statement such as the problem-etiology-signs and symptoms (PES) format includes which of the following? 1. Decreases the cost of healthcare. 2. Improves communication between nurse and client. 3. Helps the nurse focus on health and wellness elements. 4. Standardizes organization of client data.

Answer: 4. Rationale: The PES format assists with comprehensive and accurate organization of client data. More efficient planning may or may not reduce healthcare costs. Nursing diagnostic statements should be confirmed with the client but using PES does not ensure this. PES statements can be wellness or illness focused

General survey factors

Appearance, mental status , vital signs, height and weight,

When the client resists taking a liquid medication that is essential to treatment, the nurse demonstrates critical thinking by doing which of the following first? A. Omitting this dose of medication and waiting until the client is more cooperative B. Suggesting the medication can be diluted in a beverage C. Asking the nurse manager about how to approach the situation D. Notifying the physician inability to give the client this medication

B. Suggesting the medication can be diluted in a beverage Rationale: Diluting the medication in a beverage may make the medication more palatable. Using critical thinking skills, the nurse should try to problem-solve in a situation such as this before asking for the assistance of the nurse manager. Suggesting an alternative method of taking the medication (provided that there are no contraindications to diluting the medication) should improve the likelihood of the client taking the medication.

The nurse is measuring the client's urine output and straining the urine to assess for stones. Which of the following should the nurse record as objective data? A. The client reports abdominal pain B. The client's urine output was 450 mL C. The client states, "I didn't see any stones in my urine." D. The client states, "I feel like I have passed a stone."

B. The client's urine output was 450 mL. Rationale: Objective data is measurable data that can be seen, heard, or verified by the nurse. The objective data is the measurement of the urine output. A client's statements and reports of symptoms are documented as subjective data, such as the data found in options 1, 3, and 4.

How often should colorectal screenings be done and what are the different options in testing?

Beginning at age 45 until 75 y/o; -fecal occult blood test -fecal immunochemical test annually -Stool DNA test every 3 years -Flexible sigmoidoscopy every 5 years -Colonoscopy every 10 years -CT colonography every 5 years

Which professionally appropriate response should the nurse make when a more stringent policy for the use of restraints is introduced on a surgical unit? A. Use the previous, less restrictive policy conscientiously B. Express immediate disagreement with the new policy C. Ask for the rationale behind the new policy D. Obey the policy but continue to voice disapproval of it to co-workers

C. Ask for the rationale behind the new policy Rationale: Understanding the rationale behind a decision helps the nurse analyze the proposed change and understand its purpose. Options 1, 2, and 4 represent unprofessional behavior. Option 1 also places a client's safety at risk.

Which of the following outcome goals has the nurse designed correctly for the postoperative client's plan of care? Select all that apply. A. Client will state pain is less than or equal to 3 on zero to ten pain scale B. Client will have no pain C. Client will state pain is less than or equal to a 3 on a 0-10 pain scale within 24 hours D. Client will state pain is less than or equal to a 5 on a 0-10 pain scale by the time of discharge E. Client will be medicated every 4 hours by the nurse

C. Client will state pain is less than or equal to a 3 on a 0-10 pain scale within 24 hours D. Client will state pain is less than or equal to a 5 on a 0-10 pain scale by the time of discharge Rationale: An outcome goal should be SMART: specific, measurable, appropriate, realistic, and timely. Options 3 and 4 are SMART goals. Options 1 and 2 have no timeframe to achieve the goal and are therefore incomplete. Option 2 is also unrealistic; the nurse cannot expect a postoperative client to be pain free. Option 5 is not a client goal.

In developing a plan of care for a client with chronic hypertension, which nursing activity would be most important? A. Set incremental goals for blood pressure reduction B. Instruct the client to make dietary changes by reducing sodium intake C. Include the client and family when setting goals and formulating the plan of care D. Assess past compliance to medication regimens

C. Include the client and family when setting goals and formulating the plan of care Rationale: In developing a plan of care, nurses engage in a partnership with the client and family. Nurses do not plan care for clients; instead they plan care with clients and families. Assessment (option 4), goal setting (option 1), and interventions (option 2) will be most accurate and effective when carried out in partnership with the client and family. The other options represent other actions to take, but they will have less overall effectiveness if the client and family are not part of the plan.

The nurse feels a client is at risk for skin breakdown because he has only had clear liquids for the last 10 days (and essentially no protein intake). The nurse would formulate which diagnostic statement that would best reflect this problem? A. Risk for malnutrition related to clear liquid diet B. Impaired skin integrity related to no protein intake C. Risk for impaired skin integrity related to malnutrition D. Impaired nutrition related to current illness

C. Risk for impaired skin integrity related to malnutrition Rationale: This is a risk diagnosis, and the diagnostic statement has two parts: the human response (impaired skin integrity) and the related/risk factor (malnutrition). Options 1 and 2 do not have related factors that are under the control of the nurse (i.e., type of diet ordered). The diagnosis in option 4 does not specify the type of impairment (greater than or less than body requirements) and is therefore incomplete. It also does not provide direction for development of goals and interventions.

Which nurse is demonstrating the assessment phase of the nursing process? A.The nurse who observes that the client's pain was relieved with pain medication B. The nurse who turns the client to a more comfortable position C. The nurse who ask the client how much lunch he or she ate D. The nurse who works with the client to set desired outcome goals

C. The nurse who ask the client how much lunch he or she ate Rationale: Assessment involves collecting, organizing, validating, and documenting data about a client. Option 1 represents the evaluation phase. Option 2 represents the implemention phase. Option 4 represents the planning phase.

A client in the emergency department has a non-life-threatening wound. The unit is busy with other clients, families, and people in the waiting room. How should the nurse conduct an interview with this client? 1. Have the client wait until the department quiets down, as the wound is not too serious. 2. Tell the client to wait in the waiting room and fill out the paperwork. 3. Draw curtains around the client and nurse to provide as much privacy as possible. 4. Make sure the client's back is to the rest of the room so as not to be heard by passersby.

Correct Answer: 3 Rationale 1: Having the client wait may cause an unnecessary delay in treatment. Rationale 2: Having the client wait and fill out paperwork may cause an unnecessary delay in treatment. Rationale 3: The interview setting should be in a well-lighted, well-ventilated room that is relatively free of noise, movements, and distractions in order to encourage communication. The interview should also take place in an area where others cannot overhear or see the client if possible. In this situation, at least pulling a privacy curtain will help keep the client from view of others in the department. Rationale 4: Making sure the clients back is to the rest of the room is not acceptable.

In giving a change-of-shift report, which type of client information communicated by the nurse is most appropriate? A. Vital signs are stable B. Client is pleasant, alert, and oriented to time, place, and person C. The chest x-ray results were negative D. Client voided 250 mL of urine 2 hours after the urinary catheter removal

D. Client voided 250 mL of urine 2 hours after the urinary catheter removal Rationale: A change-of-shift report should include significant changes (good or bad) in a client's condition. The information should be accurate, concise, clear, and complete. Options 1 is vague and options 2 and 3 are normal data and are therefore of lesser importance to convey in the change-of-shift report.

Twenty minutes after administering pain medication to the client, the nurse returns to ask if the client's level of pain has decreased. The nurse documents the client's response as part of which phase of the nursing process? A. Diagnosis B. Planning C. Implementation D. Evaluation

D. Evaluation Rationale: Evaluating is the process of comparing client responses to the outcome goals to determine whether, or to what degree, goals have been met. Diagnosing identifies health problems, risks, and strengths. Planning is the formulation of client goals and nursing strategies (interventions) required to prevent, reduce, or eliminate the client's health problems. Implementing is carrying out or delegating the nursing interventions.

The nurse has documented the following outcome goal in the care plan: "The client will transfer from bed to chair with two-person assist." The charge nurse tells the nurse to add which of the following to complete the goal? A. Client behavior B. Conditions or modifiers C. Performance criteria D. Target time

D. Target time Rationale: The outcome goal does not state the target timeframe for when the nurse should expect to see the client behavior ("transfer"). The condition or modifier is present ("with two assists"). The performance criterion is "from bed to chair."

Pallor

Extreme or unnatural paleness

Cranial Nerve IX

Glossopharyngeal - taste & senses carotid blood pressure

Cranial Nerve XII

Hypoglossal (tongue movement)

In assessing a patient's major risk factors for heart disease, which would the nurse want to include when taking a history? a. Family history, hypertension, stress, and age b. Personality type, high cholesterol, diabetes, and smoking c. Smoking, hypertension, obesity, diabetes, and high cholesterol d. Alcohol consumption, obesity, diabetes, stress, and high cholesterol

c) Smoking, hypertension, obesity, diabetes, high cholesterol

Skin Assessment

color, moisture, temperature, texture, turgor, vascularity, edema, lesions

Pustule

raised spot on the skin containing pus

Cyst

sac containing fluid

Papule

small, solid, raised lesion on surface of the skin

Health assessment also should consider what two other factors?

(1) The usual and expected *developmental tasks* for each age group (2) the *cultural beliefs and practices* of different people. Obtaining a heritage assessment helps gather data that are accurate and meaningful and can guide culturally sensitive and appropriate care.

Atrophy

(n.) the wasting away of a body organ or tissue; any progressive decline or failure; (v.) to waste away

A client has been admitted for acute dehydration, secondary to nausea and diarrhea. When is the best time for the nurse to conduct this client's interview? 1. As soon as the client gets to the floor 2. After the client has settled in and been oriented to the room 3. When the family is available to help 4. After the client has been medicated

2. After the client has settled in and been oriented to the room

During an initial interview, the client says, "I don't understand why I have to have surgery; I'm really not that sick or in pain right now." How should the nurse respond to the client? 1. "It's OK to be worried. Surgery is a big step." 2. "What kind of questions do you have about your surgery?" 3. "I think these are things you should be asking your doctor." 4. "Have you had surgery before?"

2. What kind of questions do you have about your surgery?

A newly admitted client is angry because nursing staff continue to ask the same questions. What should the nurse respond to this client? 1. "In order to make sure all of your information is complete, I need to ask these questions." 2. "You're right. Let me know if there's anything you need right now." 3. "I'll be done shortly, just give me a few more minutes." 4. "You shouldn't be upset. We're only doing our jobs."

2. Youre right. Let me know if theres anything you need right now.

The nurse is assessing a client's level of pain. Which open-ended question should the nurse use for this situation? 1. "Is your pain worse at night?" 2. "What brought you to the clinic?" 3. "How has the pain impacted your life?" 4. "You're feeling down about having pain, aren't you?"

3. How has the pain impacted your life?

Dorsal Recumbent

A back-lying position with knees flexed and hips externally rotated; small pillow under the head; soles of feet on the surface. Area Assessed: female genitals, female reproductive tract

Supine (horizontal recumbent)

A back-lying position with legs extended; with or without pillow under the head. Area Assessed: Head, neck, posterior and anterior thorax, lungs, breasts, axillae, heart, vital signs, abdomen, upper and lower extremity, peripheral pulses

Sitting

A seated position, back unsupported and legs hanging freely. Area Assessed: Head, neck, posterior and anterior thorax, lungs, breasts, axillae, heart, vital signs, upper and lower extremity, reflexes

Which of the following behaviors by the nurse demonstrates that the nurse is participating in critical thinking? Select all that apply. A. Admitting not knowing how to do a procedure and requesting help B. Using clever and persuasive remarks to support an opinion or position C. Accepting without question the values acquired in nursing school D. Finding a quick and logical answer, even to complex questions E. Gathering three assistants to transfer the client to a stretcher after noting the client weighs 300 lbs.

A. Admitting not knowing how to do a procedure and requesting help E. Gathering three assistants to transfer the client to a stretcher after noting the client weighs 300 lbs. Rationale: Critical thinking in nursing is self-directed, supporting what nurses know and making clear what they do not know. It is important for nurses to recognize when they lack the knowledge they need to provide safe care for a client (option 1). Nurses must also utilize their resources to acquire the support they need to care for a client safely (option 5). Options 2, 3, and 4 do not demonstrate critical thinking.

The nurse assigned to care for a postoperative client has asked an unlicensed assistive person (UAP) to help the client ambulate in the hall. Before delegating this task, the nurse must do which of the following? A. Assess the client to be sure ambulation with assistance is an appropriate care measure B. Ask the client if he or she is ready to ambulate C. Ask whether the UAP has time to assist the client D. Ask the charge nurse whether UAPs have ambulated the client during this shift

A. Assess the client to be sure ambulation with assistance is an appropriate care measure Rationale: Prior to delegating any client care responsibilities, the nurse must assess the client to assure that the delegation is appropriate to his or her care. Options 2, 3, and 4 would not constitute an assessment of the client's current status.

When evaluating an elderly client's blood pressure (BP) of 146/78 mmHg, the nurse does which of the following before determining whether the BP is normal or represents hypertension? A. Compare this reading against defined standards B. Compare the reading with one taken in the opposite arm C. Determine gaps in the vital signs in the client record D. Compare the current measurement with previous ones

A. Compare this reading against defined Rationale: Analysis of the client's BP requires knowledge of the normal BP range for an older adult. The nurse compares the client's data against identified standards to determine whether this reading is normal or abnormal. Measuring the BP in the other arm (option 2) and comparing the reading to previous ones (option 4) will give additional client data, but the comparison alone will not determine whether the BP is normal. Gaps in the record (option 3) will not aid in interpreting the current measurement.

The client states, "My chest hurts and my left arm feels numb." The nurse interprets that this data is of which type and source? A. Subjective data from a primary source B. Subjective data from a secondary source C. Objective data from a primary source D. Objective data from a secondary source

A. Subjective data from a primary source Rationale: The client states, "My chest hurts and my left arm feels numb." The nurse interprets that this data is of which type and source?

Which activity would be appropriate for the nurse to delegate to an unlicensed assistive person (UAP)? A. Taking vital signs of clients on the nursing unit B. Assisting the physician with an invasive procedure C. Adjusting the rate on an infusion pump D. Evaluating achievement of client outcome goals

A. Taking vital signs of clients on the nursing unit Rationale: Part of the professional nurse's role is to delegate responsibility for activities while maintaining accountability. The nurse must match the needs of the client with the skills and knowledge of UAPs. Certain skills and activities, such as those in options 2, 3, and 4, are not within the legal scope of practice for a UAP.

Upon entering a room, a client and spouse are found crying. The nurse decides to sit with both of them, offering presence and listening to their fears instead of providing the planned education. What action did the nurse perform? 1. Implementing nursing intervention 2. Determining the nurse's need for assistance 3. Supervising delegated care 4. Reassessing the client

4. Reassessing the client

The nurse is preparing to administer a medication that the agency designates as "high alert." What action should the nurse take? 1. Ask another registered nurse to verify the medication. 2. Call the pharmacist to check the efficacy of the medication. 3. Decline to administer the medication unless there is a physician present. 4. Request that the nursing supervisor administer the medication.

Answer: 1

The nurse is preparing to conduct a mental status assessment. What should the nurse include in this assessment? 1. Cognitive and affective functions 2. Cognitive and effective functions 3. Affective and memory functions 4. Affective and knowledge functions

Answer: 1

The nurse is to administer a tuberculin test to a client who is 6 feet tall and weighs 180 pounds. Which is the most appropriate for the nurse to use? 1. A tuberculin syringe, #25-#27 gauge, 1/4- to 5/8-inch needle 2. Two 3-mL syringes, #20-#23 gauge, 1 1/2-inch needle 3. 2-mL syringe, #25 gauge, 5/8-inch needle 4. 2-mL syringe, #20-#23 gauge, 1-inch needle

Answer: 1

The nurse is working with an elderly male client on a medical unit. Which statement demonstrates elderspeak by the nurse? 1. "It's time for us to go to physical therapy." 2. "I think it would be better if you were planning to go to a nursing home after discharge." 3. "Your children must really love their dad." 4. "Your wife must be having trouble adjusting to your illness."

Answer: 1

The nurse observes during a dressing change that the client's wound has become infected. When asked by the client how the wound looks, the nurse says "it looks fine" but the nurse's facial expression does not support the response. Which aspect of communication should this nurse improve? 1. Adaptability 2. Credibility 3. Timing and relevance 4. Clarity and brevity

Answer: 1

The nurse wants to gain information about a client's situation. Which question should the nurse use to maximize communication with this patient? 1. "What brings you to the hospital?" 2. "Are you having pain?" 3. "Does your pain feel better or worse today?" 4. "Is there anything I can do for you?"

Answer: 1

When attempting to locate recent laboratory results, the nurse notices that each department has a separate section in the client's chart. Which type of documentation system is the nurse using? 1. Source-oriented record 2. Problem-oriented record 3. Case management 4. Focus charting

Answer: 1

While preparing to administer an eye ointment, the nurse inadvertently squeezes the tube, discarding the first bead of medication. What action should the nurse take at this point? 1. Administer the eye ointment as ordered, as the first bead of ointment should be discarded anyway. 2. Notify the pharmacy and request a new, unopened tube of ointment. 3. Have a second licensed nurse witness the waste and sign the chart. 4. Continue to squeeze the tube until a clear line of ointment has been discarded from the tip.

Answer: 1

A client who has been sullen and withdrawn since receiving the news of having cancer asks for assistance with a shower. Which comment by the nurse is the most appropriate? 1. "If you look better, you might feel better." 2. "Taking a shower might wash away some of that gloom and doom." 3. "This is a positive sign. I'll be right back with your supplies." 4. "Your spouse will be glad to see that you're feeling better."

Answer: 3

A client with terminal cancer has this nursing diagnosis: Pain related to neuromuscular involvement of disease process. The goal statement is: Client will be free of pain within 48 hours. As an intervention, the nurse will administer narcotic analgesics and titrate to an appropriate level. What is the flaw in this plan? 1. The goal statement is written inaccurately. 2. The interventions are dependent of nursing. 3. The goal is unrealistic. 4. The interventions are not clear enough.

Answer: 3

The nurse is preparing to administer eardrops to a 6-year-old client. What nursing action is correct? 1. Pull the earlobe down and back to straighten the ear canal. 2. Insert the tip of the applicator into the ear canal. 3. Put the eardrops in the refrigerator for 10 minutes prior to administration. 4. Press gently on the tragus of the ear a few times after administration.

Answer: 4

The nurse is to administer 0.5 mL of a medication by intramuscular injection to an older emaciated client. Which is the most appropriate for the nurse to use? 1. A tuberculin syringe, #25-#27 gauge, 1/4- to 5/8-inch needle 2. Two 3-mL syringes, #20-#23 gauge, 1 1/2-inch needle 3. 2-mL syringe, #25 gauge, 5/8-inch needle 4. 2-mL syringe, #20-#23 gauge, 1-inch needle

Answer: 4

What are risk factors that can affect liver function?

-Alcoholism -Obesity -Type 2 diabetes -injecting drugs

abnormal respiratory sounds

-Rales -wheezing -rhonchi -stridor -fine crackles -course crackle

abnormal respiratory patterns

- Tachypnea - Bradypnea - Hyperventilation - Hypoventilation - Cheyne-Stokes

A client in long-term care is scheduled for a review of the assessment and care screening process. Where should the nurse document this information? 1. MDS 2. OBRA 3. CBE 4. Kardex

1. MDS

An experienced nurse has just walked into the room of a newly assigned client. Which observation should the nurse use to include a new nursing diagnosis in this client's plan of care? 1. The client's eyes are closed. 2. The client's skin is pale and mottled. 3. The client's spouse is asleep in the chair next to the bed. 4. The television is on and the volume is turned up.

2. The clients skin is pale and mottled.

A nurse has been assigned a new client who cannot speak English. How should the nurse facilitate communication with this client? 1. Have a member of the housekeeping staff who speaks the same language translate. 2. Use the translation services supplied by the hospital. 3. Make sure a family member who does speak English is available. 4. Conduct the interview using hand gestures.

2. Use the translation services supplied by the hospital.

A client has been having pain without any clear pathology for cause. Which nursing diagnosis should the nurse identify as being the most appropriate for this client? 1. Pain due to unknown factors 2. Pain related to unknown etiology 3. Pain caused by psychosomatic condition 4. Pain manifested by client's report

2. Pain related to unknown etiology

The nurse is caring for a new mother and infant. Which action should the nurse take that allows the new parents to feel in control when being taught how to bathe their infant? 1. Telling the parents everything the nurse is doing and why 2. Letting the parents watch a video after the bath 3. Letting the parents bathe the baby with direction and guidance from the nurse 4. Giving lots of advice and suggestions about different methods

3. Letting the parents bathe the baby with direction and guidance from the nurse

The nurse is reviewing a client's chart in a facility that utilizes problem-oriented recording. In which section would the nurse find the most recent physician orders? 1. Database 2. Problem list 3. Plan of care 4. Progress notes

3. Plan of care

A nurse in the intensive care unit consults unit policy and administers a routinely used medication to a client admitted to the unit with severe hypotension. What did the nurse implement in this situation? 1. A STAT order 2. A one-time order 3. A prn order 4. A standing order

4. A standing order

Before providing care, the nurse reviews the client's pertinent history, daily treatments, diagnostic procedures, allergies, problems, and other information. Which form should the nurse review to learn all of this information? 1. The client's medical record 2. The MAR (medication administration record) 3. The written care plan 4. The Kardex

4. The Kardex

The nurse is reviewing the difference between evaluation and assessment with a new graduate nurse. What should the nurse emphasize as the major difference between these two steps in the nursing process? 1. Assessment is done at the beginning of the process. 2. Evaluation is completed at the end of the process. 3. They are the same and there is no need to differentiate. 4. The difference is in how the data are used.

4. The difference is in how the data are used.

During an assessment interview, the client states that an elective surgical procedure will not be done because it does not fit into the client's life goals. Into which of Gordon's functional health patterns should the nurse identify this client's comment? 1. Cognitive/perceptual pattern 2. Coping/stress-tolerance pattern 3. Health-perception/health-management pattern 4. Value/belief pattern

4. Value/belief pattern

The nurse is preparing to assess a client's reflexes. What equipment should the nurse gather before entering the room? 1. Sterile gloves 2. Clean gloves 3. Percussion hammer 4. Penlight

Answer : 3

A primary care provider writes a prescription for 0.15 milligram of digoxin intravenously every day. The medication is available in a concentration of 400 micrograms per mL. How many mL will the nurse administer?

Answer: 0.375 or rounded to 0.38 mL

Zone of proximity: public

Communication when speaking to an audience or small groups

Kardex

Concise method of organizing and recording data about a client making information quickly accesible to all health professionals

Zone of proximity: Personal

Distance when interacting with close friends

standardized care plans (standards of care)

Is a formal plan that specifies the nursing care for groups of clients with common needs. (All clients with myocardial infarction)

Which would be absorbed the fastest: pill, capsule or liquid?

Liquid

Single order

Medication that is to be given only once, and at a specific time, such as a preoperative order.

Cranial Nerve III

Oculomotor (motor)

Cranial Nerve I

Olfactory (smell)

Why would the very young and very old clients need to be closely monitored by the nurse for signs and symptoms of drug toxicity?

Problems with decreased body size, altered body composition, decreased kidney and liver functions cause many drugs to accumulate in older peoples bodies at dangerously high levels and for longer times.

Zone of proximity: Social

Space when interacting with acquaintances such as in a work or social setting

How often should women have a mammogram?

Start at age 45; ages 45-54 screened annually. Age 55 or older should transition to biennial

A client is suspected of having decreased liver function. What information would be essential for the nurse to know about the oral medication that is to be given?

The nurse would need to know which organ that particular drug will be processed through. If it is going to be processed through the liver and they have decreased function, this may do more harm than good for that organ.

Cranial nerve V

The trigeminal nerve, which is responsible for innervating the muscles of mastication.

PRN order

as needed order, permits the nurse to give a medication when, in the nurse's judgment, the client requires it

retrospective audit

evaluation of a client's record after discharge from an agency

standardized care plans

prepared plan of care that identifies the nursing diagnoses, patient goals, and related nursing orders common to a specific population (e.g., normal neonates) or problem

Auscultation of heart sounds

• Aortic valve: 2nd intercostal space at R of sternal border • Pulmonic valve: 2nd intercostal space at L sternal border • Erb's point: 3rd intercostal, L, sterna border. (S2) • Tricuspid Area: 4th intercostal space at the L sternal border • Mitral Area: 5th intercostal space, L, midclavicular line (s1) • Listen for extra sounds or murmurs while concentrating on systole and diastole (state if S3 and S4 are present)

Interdisciplinary team approach

• Teamwork eliminates unnecessary duplication. Appropriate duplication reinforces learning. • PT is responsible for gait training, OT incorporates, carries over gains made in PT • OT focuses on functional mobility, incorporating mobility aids

S3 heart sound

↑ ventricular filling pressure (e.g., mitral regurgitation, HF), common in dilated ventricles

Individualized care plan

a plan tailored to meet the unique needs of a specific client--needs that are not addressed by the standardized plan

Sims position

a side-lying position with the lowermost arm behind the body and the uppermost leg flexed at hip and knee, upper arm flexed at shoulder and elbow Area Assessed: Rectum , vagina

Plaque

a solid mass on the surface of the skin

Informal nursing care plan

a strategy for action that exists in the nurse's mind

The client states: "I really don't want anyone to visit me who has not been cleared by me first." If utilizing the SOAP format, in which category should the nurse document this statement? 1. Subjective data 2. Objective data 3. Assessment 4. Planning

1. Subjective data

A home care client must correctly self-administer insulin injections before being discharged from the agency. On what skill is this client being evaluated? 1. Technical 2. Cognitive 3. Interpersonal 4. Academic

1. Technical

standing order

A standing order may be carried out indefinitely until an order is written to cancel it or may be carried out for a specified number of days.

A cardiac specialty hospital has several written plans in place for clients who are admitted, according to specific medical diagnoses and nursing interventions. Typical nursing diagnoses as well as standard nursing interventions are included in these plans. Which type of form is this hospital utilizing? 1. Standardized care plans 2. Traditional care plans 3. Critical pathways 4. Kardex

1. Standardized care plans

steps for administering medication

*permission from medical control 1) Verify the patient 2) Verify the proper medication/prescription 3) Verify the dose 4) Verify the route 5) Check the expiration date 6) Verify the time & document 7)Check MAR x3

One of the interventions for a client with a nursing diagnosis of Impaired swallowing is to position the client upright in a chair (60-90 degrees) during feeding times. What should the nurse identify as the modifier in this intervention? 1. 60-90 degrees during feeding times 2. Position in chair 3. Upright in a chair 4. Impaired swallowing

1. 60 to 90 degrees during feeding times

The nurse is providing care to an assigned client. Which action indicates that the nurse supports the client's respect for dignity? 1. Allowing the client to complete hygienic care when possible 2. Providing all care to the client whenever possible 3. Telling the other staff that the client is demanding, so they are able to meet the client's needs 4. Presenting information to the client's family about the client's condition

1. Allowing the client to complete hygienic care when possible

On one of the first days working alone, the new nurse with limited patient teaching experience needs to instruct tracheostomy care to a client and spouse. What action should the nurse take? 1. Ask the nurse mentor to assist with the teaching after reviewing the procedure. 2. Read the policy and procedure manual before the teaching session. 3. Do the best the nurse can by remembering what was taught in nursing school. 4. Ask for a different assignment until the nurse feels comfortable with this one.

1. Ask the nurse mentor to assist with the teaching after reviewing the procedure.

A discharge goal for a client is to have improved mobility. Which outcome statement did the nurse write appropriately? 1. Client will ambulate without a walker by 6 weeks. 2. Client will ambulate freely in house. 3. Client will not fall. 4. Client will have freer movement in daily activities.

1. Client will ambulate without a walker by 6 weeks.

The nurse is completing a health history with a client who has complications from chronic asthma. Which open-ended question should the nurse use? 1. "How would you describe your sleep pattern?" 2. "Can you describe your coughing pattern?" 3. "Is there anything that makes your breathing worse?" 4. "What medications are you on?"

1. How would you describe your sleep pattern?

The nurse has just completed an admission interview with a new client. Which nursing statement indicates that the interview is in the closing phase? 1. "I'm going to set up your physical assessment now. Do you have any questions?" 2. "Tell me more about how you feel." 3. "Could you give examples of what types of other treatments you've had?" 4. "Is there anything you're worried about?"

1. Im going to set up your physical assessment now. Do you have any questions?

The nurse is teaching medication administration to a client being discharged. Which instruction should the nurse rewrite for this client? 1. Lasix, 20 mg, po bid 2. Lasix, 20 mg tablet, twice daily 3. Lasix, 20 mg by mouth, two times a day a day 4. Lasix, 20 mg by mouth 8 AM and 2 PM

1. Lasix, 20 mg, po bid

A hospital is not able to be reimbursed for care a particular client received while in the emergency department. The client came in with chest pain, which was later diagnosed as gastric reflux. Which problem with documentation might have caused the lack of reimbursement? 1. The client's record contained an incorrect DRG. 2. The client was charged for an ECG. 3. A code cart was opened and the client was charged for medications opened but not used. 4. The physician made a diagnostic mistake.

1. The clients record contained an incorrect DRG.

After an assessment, the nurse reviews the list of client problems. For which problems should the nurse create nursing diagnoses? 1. The ones that the nurse is licensed to treat 2. The ones that address other health professionals' interventions 3. The ones that focus on the client's primary illness 4. The ones that have standardized care available

1. The ones that the nurse is licensed to treat

The nurse provides care to clients admitted to a mental health facility who exhibit paranoid behavior. Which skill should the nurse use when caring for these clients? 1. Cognitive 2. Interpersonal 3. Technical 4. Therapeutic

2) interpersonal

A client did not meet the goal of walking unassisted, without assistive devices, by discharge from rehabilitation. The case manager using a critical pathway should identify this outcome as being which of the following? 1. An unattainable goal 2. A variance 3. An error in care planning 4. An error in intervention implementation

2. A variance

The nurse formulates the nursing diagnosis of Acute pain, related to tissue damage, secondary to infarction, manifested by pallor, client report, and shallow, rapid breathing for a client experiencing an acute myocardial infarction. Which collaborative action would be appropriate for this client? 1. Provide a calm, quiet atmosphere in the client's room. 2. Administer pain medication. 3. Educate the client and family regarding treatment and therapies. 4. Monitor for changes in the client's condition.

2. Administer pain medication.

The nurse wants to create an intervention to assist a client with ambulation. Which statement is the most appropriate manner for the nurse to write this intervention? 1. Assist client with ambulation. 2. Ambulate with client, using a gait belt, twice daily for 15 minutes. 3. Make sure client understands the rationale for using the gait belt. 4. Client will ambulate in hallway twice daily.

2. Ambulate with client, using a gait belt, twice daily for 15 minutes.

A client is prescribed a medication that the nurse has never administered and information about the medication is not in the drug reference manual. What should the nurse do? 1. Follow the physician's orders as written and give the medication. 2. Call the pharmacy and do further investigating before administering the medication. 3. Ask the client about this medication. 4. Call the physician and ask what the medication is and what it is for.

2. Call the pharmacy and do further investigating before administering the medication. Rationale 2: The nurse should clearly understand all nursing interventions to be implemented and question any that are not understood. The nurse is responsible for intelligent implementation of medical and surgical plans of care. The pharmacist would be the most appropriate reference point for this nurse to begin to research this problem.

The nurse administered analgesic medications to an assigned client via central line. In which section of PIE charting should the nurse document this information? 1. Plan 2. Intervention 3. Evaluation 4. Progress notes

2. Intervention

The nurse notes that a client has the outcome goal "Client will have a decrease in pain level (down to a 3) within 45 minutes of receiving oral analgesic." Which client statement should the nurse use to evaluate this goal? 1. "I'm getting really sleepy from that medication. I think I'll take a nap." 2. "My pain is a 4." 3. "I still have some pain." 4. "Will the pain ever go away?"

2. My pain is a 4.

A client has specific cultural needs that affect the plan of care. In which part of the client's problem-oriented medical record should the nurse document this information? 1. Database 2. Problem list 3. Plan of care 4. Progress notes

2. Problem list

The neonatal intensive care nurse implements several actions to prevent further complications in a newly admitted premature infant. Which type of document did the nurse use to find these actions? 1. Standardized care plan 2. Protocol 3. Standards of care 4. Policy and procedure manual

2. Protocol

A hospital is implementing the use of the NIC (Nursing Interventions Classification) taxonomy. What purpose will the implementation of this taxonomy serve? 1. Help the nurse with documentation of the care plan 2. Require that the nurse use sound judgment and knowledge of the client 3. Match nursing diagnoses to exact interventions 4. Help the nurse choose activities that are individualized to the client

2. Require that the nurse use sound judgment and knowledge of the client

The nurse provides routine morning care to a client, including all the medications and scheduled treatments. What action should the nurse make next? 1. Move on to the next assignment to increase the nurse's efficiency. 2. Report this to the charge nurse. 3. Document all care in the progress notes. 4. Get supplies organized for the next client's medications and treatments.

3. Document all care in the progress notes.

While preparing a client for a procedure, the nurse notes that the client has become unresponsive and respirations have become shallow. What type of assessment should the nurse complete at this time? 1. Initial assessment 2. Problem-focused assessment 3. Emergency assessment 4. Time-lapsed assessment

3. Emergency assessment

The nurse decides to seek wound care alternatives for a client's stasis ulcer that is not healing after treatment for 2 weeks. In which phase of the nursing process is the nurse functioning? 1. Diagnosis 2. Implementation 3. Evaluation 4. Assessment

3. Evaluation

A client is admitted to a comprehensive rehabilitation center for continuing care following a motor vehicle crash. The admitting nurse will develop the initial plan of care, but who will be involved with the ongoing planning of this client's care? 1. The admitting nurse 2. All nurses who work with the client 3. Everybody involved in this client's care 4. The client and the client's support system

3. Everybody involved in this clients care

A client has the goal statement "Client will be able to state two positive aspects of rehab therapy by the end of the week." What statement demonstrates that the nurse appropriately evaluated this goal? 1. Goal not met, client able to state one positive aspect by the end of the week. 2. Goal met, client able to state one positive aspect by the end of the week. 3. Goal met, client able to state two positive aspects of therapy by week's end. 4. Goal incomplete, client not able to positively state anything about rehab.

3. Goal met, client able to state two positive aspects of therapy by weeks end.

The nurse is reviewing the Nursing Outcomes Classification (NOC) taxonomy system. To what can the nurse compare this taxonomy? 1. Nursing diagnosis statement 2. Planning portion of the care plan 3. Goal statement of the traditional care plan 4. Implementation phase of the care plan

3. Goal statement of the traditional care plan

The nurse suspects that a client with a history of injuries is a victim of abuse. What did the nurse use to come to this conclusion? 1. Observation of cues 2. Validation 3. Inference 4. Judgment

3. Inference

The nurse is reviewing information about the formulation of nursing diagnoses. What should the nurse identify as the area in which nursing diagnoses differ from medical diagnoses and collaborative problems? 1. Mental status of the client 2. Chronic nature of the illness 3. Nursing care focus 4. Prognosis

3. Nursing care focus

A nurse is caring for a client who has a diagnosis of Impaired skin integrity, related to immobility, secondary to neurologic dysfunction. Which should the nurse identify as an observation intervention? 1. Turn and reposition client every 2 hours. 2. Cushion bony prominences with soft foam while in bed. 3. Provide ongoing assessment for skin breakdown every shift. 4. Apply lotion to dry skin twice daily.

3. Provide ongoing assessment for skin breakdown every shift.

A client is scheduled for elective hip replacement and will be admitted postoperatively to the orthopedic unit for care. What should the nurses use to help plan this client's care? 1. Informal nursing care plan 2. Formal nursing care plan 3. Standardized care plan 4. Individualized care plan

3. Standardized care plan

The graduate nurse is struggling with identifying cues from clustered data. What should the nurse use to recognize data patterns and cues? 1. Depend on knowledge gained from peers' experiences. 2. Work with seasoned and experienced nurses and learn from them. 3. Take assessment notes and utilize information from textbooks for comparison. 4. Know that this will take time, and experience is the best teacher.

3. Take assessment notes and utilize information from textbooks for comparison.

The nurse working in a hospital that utilizes a charting by exception (CBE) documentation system notes that a client did not require care in all of the areas identified on a flow sheet. What action should the nurse take? 1. Leave the areas blank. 2. Leave the areas blank, but then add an extensive explanation in the progress notes section of the chart. 3. Write N/A on the flow sheet in the areas that are not applicable to that client. 4. Make sure this information gets passed along in the shift report.

3. Write N/A on the flow sheet in the areas that are not applicable to that client.

The nurse is reviewing interventions written for a client's plan of care. Which intervention should the nurse recognize as being dependent? 1. Repositioning the client every 2 hours 2. Assisting the client with transfers to the bathroom 3. Providing ongoing physical assessment, especially of the incisional sites 4. Administering medications for pain

4. Administering medications for pain

The nurse is formulating a nursing diagnosis for a client with a long, extensive history of psychiatric problems, beginning in childhood, who is being placed in a long-term, structured institutional environment. Which diagnosis indicates the client's problem is adequately described? 1. Chronic low self-esteem, related to factors too numerous to mention 2. Risk for self-harm, related to many psychiatric problems 3. Impaired social interaction, due to long history of institutionalization 4. Alteration in thought processes, related to complex factors

4. Alteration in thought processes, related to complex factors

The nurse is documenting client care on flow sheets that identify abnormal assessment findings. Which type of documentation system is the nurse using? 1. Computerized documentation 2. Focus charting 3. SOAP charting 4. Charting by exception

4. Charting by exception

The home health nurse uses creativity and critical thinking to devise a way for a client to receive intravenous medication while sitting outside on the porch. Which skill did the nurse use for this situation? 1. Technical 2. Interpersonal 3. Creativity 4. Cognitive

4. Cognitive

The nurse has completed the initial assessment of a client and has analyzed and clustered the data. What should the nurse complete next in the diagnostic process? 1. Formulate a diagnosis. 2. Verify the data. 3. Research collaborative and nursing-related interventions. 4. Identify the client's problem, health risks, and strengths.

4. Identify the clients problem, health risks, and strengths.

A client is admitted for complications following a routine diagnostic procedure of the colon. Which type of care plan will most likely be implemented for this client? 1. Informal nursing care plan 2. Formal nursing care plan 3. Standardized care plan 4. Individualized care plan

4. Individualized care plan

The nurse documents: "Client avoids eye contact and gives only vague, nonspecific answers to direct questioning by the professional staff. Is quite animated (laughs aloud, smiles, uses hand gestures) in conversation with spouse." Which method of data collection does this documentation demonstrate? 1. Examining 2. Interviewing 3. Listening 4. Observing

4. Observing

The nurse is preparing a client for an abdominal examination. What should the nurse do before beginning the examination? 1. Ask the client to urinate. 2. Ask the client to drink 8 ounces of water. 3. Assess vital signs. 4. Assess heart rate.

Answer: 1

The nurse is performing a musculoskeletal assessment on a client admitted with a possible stroke. When testing for muscle grip strength, the nurse should ask the client to perform which action? 1. Grasp the nurse's index and middle fingers while the nurse tries to pull the fingers out. 2. Hold an arm up and resist while the nurse tries to push it down. 3. Flex each arm and then try to extend it against the nurse's attempt to keep the arm in flexion. 4. Shrug the shoulders against the resistance of the nurse's hands.

Answer: 1

7. An older client with renal insufficiency is to receive a cardiac medication. Which is the nurse most likely to administer? 1. A decreased dosage 2. The standard dosage 3. An increased dosage 4. Divided dosages

Answer: 1

A client has the goal statement "Client will have clear lung sounds bilaterally within 3 days." One intervention to meet this goal is for the nurse to teach the client to cough and deep-breathe and have the client do this several times every 2 hours. At the end of the third day, the client's lungs are indeed clear. What should the nurse do to relate the intervention to the outcome? 1. Ask how many times per day the client practiced the coughing and deep-breathing exercises. 2. Tell the client that the lungs are clear. 3. Document the assessment findings to show the effectiveness of the intervention. 4. Write this evaluation statement: Goal met, lung sounds clear by third day.

Answer: 1

During administration of an intradermal injection, the nurse notices that the outline of the needle bevel is visible under the client's skin. How should the nurse proceed? 1. Recognize that this is an expected finding in a properly administered intradermal injection. 2. Withdraw the needle, prepare a new injection, and start again. 3. Insert the needle further into the skin at a deeper angle. 4. Turn the needle so that the bevel is down and inject the medication slowly, looking for development of a bleb.

Answer: 1

During an interaction between a nurse and client, the nurse conveys respect and an attitude that shows the nurse takes the client's opinions seriously. In which stage of the working relationship are the nurse and client engaged? 1. Exploring and understanding thoughts and feelings 2. Facilitating and taking action 3. Confrontation 4. Concreteness

Answer: 1

The nurse is assessing the peripheral vascular status of an older client. Which finding should the nurse consider as being normal for this client? 1. Easy to palpate upper-extremity arteries 2. Easy to palpate lower-extremity arteries 3. Reduction in the number of varicosities 4. Increase in diastolic blood pressure

Answer: 1

The nurse is assisting the physician who is preparing to test a sexually active female client for cervical cancer. What should the nurse expect the healthcare provider to perform? 1. Pap test 2. Breast exam 3. Rectal exam 4. Abdominal exam

Answer: 1

The nurse is identifying communication strategies for a client unable to speak. What would be appropriate for the client in this situation? 1. Using a picture board to facilitate communication 2. Facing the client when speaking 3. Employing an interpreter 4. Making sure that the language spoken is the client's dominant language

Answer: 1

The nurse selects the nursing diagnosis of Willingness to learn about spiritual well-being for a family. Which data cluster did the nurse use to support this diagnosis? 1. The family visits different congregations, the parents have been reflecting on their own spiritual upbringings, and the children are questioning rituals of their friends and friends' families. 2. The children attend Sunday school classes, one parent always attends services with the children, and the parents attempt interaction with congregational activities. 3. The grandparents go to weekly services and have formal interaction with clergy. 4. The children have attended private, religious schools, and the parents are involved in the school's activities.

Answer: 1 Explanation: A health promotion diagnosis relates to clients' preparedness to implement behaviors to improve their health condition. These diagnosis labels begin with the phrase Willingness to learn about the health maintenance or Willingness to change health practices. The data cluster that describes the questioning, searching, and reflecting would support an attitude of readiness.

A client who has been in a wheelchair for several years is currently experiencing problems with skin breakdown and urinary retention in addition to depression. Which diagnosis should the nurse select for this client? 1. Syndrome diagnosis 2. Risk nursing diagnosis 3. Actual diagnosis 4. Health promotion diagnosis

Answer: 1 Explanation: A syndrome diagnosis is a diagnosis that is associated with a cluster of other diagnoses (in this situation, Urinary elimination alteration, Impaired skin integrity, and Powerlessness). Risk, actual, or health promotion diagnoses would not be appropriate for this client.

The nurse is greeting a newly admitted client. What statement should the nurse make to establish rapport with this client? 1. "Hello, I'm your nurse and I'll be taking care of you today." 2. "You're lucky-there are no students on the unit today." 3. "Good morning, is there anything you need right now?" 4. "Hi. If you need anything, put on your call light."

Answer: 1 Explanation: Establishing rapport is a process of creating goodwill and trust and usually begins with a greeting and self-introduction, accompanied by nonverbal gestures such as a smile, a handshake, and a friendly manner. Making introductions, especially offering the use of your name, is especially good in establishing rapport.

The nurse has formulated a diagnosis of Activity intolerance related to decreased airway capacity for a client with chronic asthma. In looking at the client's coping skills, the nurse realizes that the client has a vast knowledge about the disease and what exacerbates symptoms in particular situations. Why should the nurse utilize this information? 1. Strengths can be an aid to mobilizing health and the healing process. 2. The client will be more active in the plan. 3. It will be easier for the nurse to educate the client about other interventions. 4. The nurse won't have to spend time going over the pathology of the client's disease.

Answer: 1 Explanation: Establishing strengths, resources, and ability to cope will help the client develop a more well-rounded self-concept and self-image. Strengths can be an aid to mobilizing health and regenerative processes.

Unlicensed assistive personnel measure a newly admitted client's vital signs to be: temperature = 99.3°F, respirations = 26, pulse = 98 bpm, and blood pressure = 200/146 mm Hg. What should the nurse do to validate this data? 1. Retake the vital signs. 2. Call the physician. 3. Continue with the physical assessment as soon as possible. 4. Report the findings to the charge nurse.

Answer: 1 Explanation: Guidelines for validating assessment data that are out of normal range include repeating the measurements, using another piece of equipment as needed to confirm abnormalities, or asking someone else to collect the same data. In this situation, the nurse needs to be sure that the vital signs are accurate.

The nurse identifies the diagnosis of Anxiety, related to unfamiliarity of disease process, manifested by restlessness and tachycardia for a client newly diagnosed with pancreatic cancer. What is the etiology of this diagnosis? 1. Unfamiliarity of disease process 2. Anxiety 3. Restlessness 4. Tachycardia

Answer: 1 Explanation: The etiology is the underlying cause and a contributing factor of the client's response. In this case, the uncertainty of the diagnosis, fear of the unknown, and response to the diagnosis cause the client to become anxious and upset. Anxiety, restlessness, and tachycardia are defining characteristics.

The nurse is reviewing a client's plan of care. Which statements indicate that this care plan has been completed accurately and appropriately? (Select all that apply.) 1. Ineffective coping related to drug abuse as evidenced by drug overdose. 2. The client will identify two healthy coping mechanisms by time of discharge. 3. The client has identified two health coping mechanisms to replace inappropriate drug use. 4. The client will be provided with guidance in identifying healthy coping mechanisms. 5. The client has apologized to his family for drug abuse behaviors.

Answer: 1, 2, 3, 4

The nurse manager observes a staff nurse perform actions within the nursing process. Which activities did the manager observe the nurse perform? (Select all that apply.) 1. Notifying the surgeon that a postoperative client is experiencing an increase in temperature 2. Advocating for a client who is mentally incapable of expressing her needs 3. Deciding to increase a client's nasal oxygen based on his current pulse oxygenation levels 4. Documenting all clients' pain level responses after the administration of pain medication 5. Attending in-services on a new hydraulic lift to be used to support safe client care

Answer: 1, 2, 3, 4

The nurse manager is implementing computerized care plans for the care area. Which guidelines should the manager emphasize when the staff is writing care plans? (Select all that apply.) 1. Plans must be dated and signed. 2. Categories must have headings. 3. Plans must be specific. 4. Plans must include preventive care and health maintenance. 5. Plans must include interventions for ongoing assessment. 6. Plans are standardized and generalized for all clients.

Answer: 1, 2, 3, 4, 5

The nurse is preparing to formulate nursing diagnoses for a client desiring information to help with chronic low back pain. Which human response patterns should the nurse keep in mind when formulating the diagnoses for this client? (Select all that apply.) 1. Moving 2. Choosing 3. Perceiving 4. Anticipating 5. Communicating

Answer: 1, 2, 3, 5

A nurse is performing an initial assessment on a new admission. What information should the nurse consider as being a part of the database? (Select all that apply.) 1. Reports from physical therapy the client received as an outpatient 2. Documentation of the nurse's physical assessment 3. Physician's orders 4. A list of current medications 5. Information about the client's cultural preferences 6. Discharge instructions

Answer: 1, 2, 4, 5

The nurse is devising a care plan for a client with complex health issues and current acute health problems. Which criteria should the nurse ensure is used when planning interventions for this client? (Select all that apply.) 1. Congruent with the client's values, beliefs, and culture 2. Are within established standards of care 3. Based on scientific and medical knowledge 4. Achievable with the resources available 5. Must be safe and appropriate for the client's age

Answer: 1, 2, 4, 5

The nurse is documenting care provided to a client. Which action should the nurse take to demonstrate the avoidance of potentially confusing abbreviations when documenting? (Select all that apply.) 1. Documenting vital signs as "TPR." 2. Charting that the "drsg was dry and intact." 3. Transcribing a verbal order as "Carbamazepine 12 mcg/ml IV push daily." 4. Documenting "Client consistently requesting IM MS for pain well before prescribed time." 5. Charting, "Client to be ambulated q.i.d."

Answer: 1, 2, 5

The nurse is reviewing the nursing process with a first-year nursing student. What should the nurse explain as being the purpose of the diagnosis phase? (Select all that apply.) 1. Develop a list of problems. 2. Identify client strengths. 3. Develop a plan. 4. Specify goals and outcomes. 5. Identify problems that can be prevented.

Answer: 1, 2, 5

The nurse has formulated a nursing diagnosis of Impaired skin integrity related to poor hygienic practice, secondary to current living conditions for a client. Which data did the nurse use to support this diagnosis? (Select all that apply.) 1. The client has dry, cracked skin. 2. The client has one large and several smaller open, ulcerated areas on his right leg. 3. The client does not drive. 4. The client states that he does not use alcohol or drugs. 5. The client's clothes are soiled. 6. The client has obvious body odor.

Answer: 1, 2, 5, 6

The nurse is reviewing assessment data collected for a client's care plan. What criteria should the nurse use when formulating this client's nursing diagnoses? (Select all that apply.) 1. Nonjudgmental statements 2. Stated in terms of a need 3. Must be legally advisable 4. Cause/effect correctly stated 5. Medical terminology used to describe the cause 6. Diagnosis worded specifically and precisely

Answer: 1, 3, 4, 6

The nurse works at an organization that is installing a new computerized record system. What should the nurse learn that has been implemented to help ensure the security of client records? (Select all that apply.) 1. A firewall to protect the server from unauthorized access 2. One unit password to protect the unit's information 3. Expectation to log off a terminal after using it 4. Expectation to turn the monitor away from view when unattended 5. Requirement to shred all computer-generated worksheets

Answer: 1, 3, 5

The primary care provider prescribed 5 mL of a medication to be given deep intramuscular for a 40-year-old female who is 5'7" tall and weighs 135 pounds. Which is the most appropriate equipment for the nurse to use? Select all that apply. 1. Two 3-mL syringes 2. One 5-mL syringe 3. A #20-#23 gauge needle 4. A 1-inch needle 5. A 1 1/2-inch needle

Answer: 1, 3, 5

The nurse is using the PES model to write a nursing diagnosis. Which nursing diagnoses demonstrate that the nurse used this model appropriately? (Select all that apply.) 1. Ineffective coping related to depression as evidenced by suicide attempt 2. Noncompliance (DASH diet) related to denial of having disease 3. Risk for infection related to recent surgery 4. Nutrition less than adequate related to anxiety as evidenced by weight loss of 10 pounds 5. Ineffective Breathing Pattern as evidenced by cyanotic lips

Answer: 1, 4

The nurse is conducting an interview with a new client. Which actions indicate that the nurse is implementing effective communication guidelines? (Select all that apply.) 1. Looking directly at the client to ensure good eye contact 2. Managing the conversation to avoid periods of silence 3. Providing personal experiences to help the client focus 4. Sitting in a chair next to the client who is in bed 5. Keeping arms unfolded and in a relaxed position

Answer: 1, 4, 5

The nurse is preparing to provide care planned for a client. What actions should the nurse complete during this phase of client care? (Select all that apply.) 1. Evaluating the outcome of the interventions 2. Reassessing the client 3. Documenting the history and physical 4. Supervising delegated care 5. Implementing the nursing interventions

Answer: 1, 4, 5

The nurse wants to adhere to practice guidelines that meet legal and ethical standards when documenting client care. Which actions should the nurse take to prove adherence? (Select all that apply.) 1. Charting the client's response to pain medication taken 2. Describing the client as "appearing to be comfortable" 3. Leaving sufficient charting space for the previous shift to chart client teaching 4. Documenting that the client reports, "I'm so afraid of tomorrow's surgery" 5. Making a late entry regarding a client's request for pain medication

Answer: 1, 4, 5

Upon aspirating a saline lock prior to administering intravenous medication, the nurse notes that there is no blood return. What nursing action should be taken? 1. Discontinue this infiltrated lock and restart another site for medication administration. 2. Slowly infuse 1 mL of saline into the lock, assessing for infiltration. 3. Reinsert the needle into the lock and aspirate using more pressure. 4. Pull the intravenous catheter out inch and attempt aspiration.

Answer: 2

The nurse is providing care to a client. Which nursing diagnoses can the nurse apply when providing client care? (Select all that apply.) 1. Ineffective Breathing Pattern 2. Risk of Infection 3. Willingness for Enhanced Nutrition 4. Willingness for Enhanced Family Coping 5. Anxiety

Answer: 1, 5

The nurse is assessing the nose and sinuses of a client. Which findings should the nurse identify as being within normal limits? (Select all that apply.) 1. Nose straight 2. Nares symmetrical 3. No tenderness over the bridge 4. Air movement restricted in one nare 5. Clear drainage from one nare

Answer: 1,2,3

The nurse is preparing to complete a physical examination on a client. What should the nurse realize as being the purpose for this examination? (Select all that apply.) 1. Obtain baseline data. 2. Obtain data to help determine nursing diagnoses. 3. Identify areas for disease prevention. 4. Identify the client's employment status. 5. Obtain data about the client's leisure activities.

Answer: 1,2,3

The nurse is utilizing the technique of inspection during a physical examination with a client. When using this technique, the nurse will take which actions? (Select all that apply.) 1. Visually observe a body area. 2. Obtain information through the sense of smell. 3. Obtain information through the sense of hearing. 4. Examine the body through the use of touch. 5. Strike the body to elicit a sound from a body part.

Answer: 1,2,3

The nurse is communicating with an older client. Which actions demonstrate that the nurse understands the best approaches to communicate with this client? (Select all that apply.) 1. Asking, "What can I do to make you feel safe?" 2. Observed intently listening to the client describe how being alone makes her feel 3. Offering to take the client "out for a walk" 4. Consistently arranging for the client to have hair done 5. Managing to get a copy of the client's favorite magazine

Answer: 1,2,5

The nurse is beginning a helping relationship with a newly admitted client. Which behaviors should the nurse demonstrate that support this type of relationship? (Select all that apply.) 1. Becoming familiar with the client's social history by reading the admission interview 2. Orienting the client to the physical layout of the facility as well as to the facility's policies 3. Gaining the client's trust by consistently keeping promises to return and "visit" 4. Respecting the client's wish to be alone after hearing about the loss of a family friend 5. Asking to remain with the client when he is experiencing symptoms of the flu

Answer: 1,3,4,5

A client tells the nurse, "This pill is a different color than the one that I usually take at home". Which is the best response by the nurse? 1. "Go ahead and take your medicine". 2. "I will recheck your medication order". 3. Maybe the doctor ordered a different medication." 4. "I'll leave the pill here while I check with the doctor."

Answer: 2

While performing a health assessment, in which position should the nurse place the client for inspection of the jugular veins? 1. 90-degree angle 2. 30- to 45-degree angle 3. 15-degree angle 4. 60-degree angle

Answer: 2

A client who is being transferred to a rehabilitation center wants to take the medical record to the new facility. How should the nurse respond to this client's request? 1. "You'll have to ask your doctor for permission to do that." 2. "Actually, the original record is the property of the hospital, but you are welcome to copies of your records." 3. "We'll make sure that all of your records are sent ahead to the rehab hospital, so you don't really have to worry about those details." 4. "There's a new law that protects your records, so you're not going to be able to have access to them."

Answer: 2

A hospitalized client is prescribed acetaminophen 325 mg 2 tablets every 4 hours prn temperature over 101°F. The client complains of a headache. Can the nurse legally administer the medication to treat the headache? 1. Yes, as acetaminophen is used both for fever and headache. 2. No, not unless the client also has a temperature over 101°F. 3. Yes, but the nurse should document the reason why the medication was administered as a temperature elevation. 4. Yes, because the medication is available over the counter, an order is not required.

Answer: 2

A nursing diagnosis of Risk for Deficient Fluid Volume related to excessive fluid loss, secondary to diarrhea and vomiting was implemented for a home health client who began with these symptoms 5 days ago. A goal was that the client's symptoms would be eliminated within 48 hours. The client is being seen after a week and has had no diarrhea or vomiting for the past 5 days. What should the nurse do? 1. Keep the problem on the care plan, in case the symptoms return. 2. Document that the problem has been resolved and discontinue the care for the problem. 3. Assume that whatever the cause was, the symptoms may return, but document that the goal was met. 4. Document that the potential problem is being prevented because the symptoms have stopped.

Answer: 2

A spouse arrives to visit a client who is currently receiving cardiopulmonary resuscitation. Which statement should the nurse make to the spouse? 1. "I know you're worried about your loved one. I'm sure this is a difficult situation for you. Do you have any questions right now?" 2. "Your spouse's heart stopped. All these people are here to help get it started." 3. "Your spouse's physician will be here shortly and explain all of the medication and treatment that your spouse is receiving right now." 4. "Is there someone you would like to call? I'm sure this is a scary situation and you may feel more comfortable if someone were with you during this time."

Answer: 2

At which point of preparing medication from an ampule does the nurse anticipate using a filter needle? 1. Filter needles are not used for this preparation. 2. When drawing the medication from the ampule. 3. When administering the medication to the client. 4. Both for drawing up the medication and for administering the medication.

Answer: 2

Proper administration of an otic medication to a 2-year-old client includes which of the following? 1. Pull the ear straight back. 2. Pull the ear down and back. 3. Pull the ear up and back. 4. Pull the ear straight upward.

Answer: 2

The client who regularly uses a metered-dose inhaler four times a day tells the nurse that it is difficult to tell when the canister is empty. What instruction should the nurse give this client? 1. Place the canister in a bowl of water. If the canister floats, it is not empty. 2. When you get a new canister, divide the number of puffs that is listed on the label by four. That will tell you how many days the canister will last. 3. You can tell that the canister is empty when you can no longer smell the medication when you activate the plunger. 4. When you feel like you are no longer getting maximum effect from the medication, your canister is empty.

Answer: 2

The nurse has just injected insulin subcutaneously into the client's abdomen. What action should the nurse take at this point? 1. Massage the site to encourage absorption. 2. Leave the needle embedded in the client's skin for 5 seconds after administration. 3. Remove the needle rapidly by pulling it quickly from the skin. 4. Cover the injection site with a pressure dressing for at least 15 minutes or until the bleb disappears.

Answer: 2

The nurse is caring for a client with Parkinson disease who desires to improve fine motor skills. Which statement should the nurse identify as an appropriate collaborative intervention for this client? 1. Provide assistance as needed with dressing and grooming. 2. Provide assistive devices and educate client to use grab bar and large-handled utensils. 3. Make sure lighting and space are adequate for client. 4. Administer medications to improve muscle tone.

Answer: 2

The nurse is planning to administer medications to a new client. What is the nurse's greatest priority in administering these medications? 1. Be certain the medications are given within 15 minutes of the time they are scheduled. 2. Before giving the medications, know what the intended effects are for this client. 3. Assess the client's knowledge of the action of the medications. 4. Document the administration accurately so the reimbursement is correct.

Answer: 2

The nurse is preparing to administer a cardiotonic drug to a client. Which assessment should the nurse perform before administering the medication? 1. Respiratory rate 2. Apical pulse 3. Popliteal pulse 4. Capillary blanch test

Answer: 2

The nurse is providing discharge teaching for a client who is prescribed a bronchodilator inhaler and a corticosteroid inhaler. What information should the nurse provide regarding the dosage schedule for these two medications? 1. Always use the corticosteroid inhaler first. 2. Use the bronchodilator first. 3. It makes no difference which inhaler is used first. 4. Use the inhalers on alternate days, not on the same day.

Answer: 2

The nurse is working in a pediatric clinic and has to explain a nebulizer treatment to a child. Which approach should the nurse use? 1. Give the child's parent a full explanation, but make sure the child hears what is said. 2. Let the child handle the equipment first, then demonstrate on the child's doll. 3. Start the treatment, but make sure that the parent is there to comfort the child if she becomes afraid. 4. Make sure that the physician is available for questions.

Answer: 2

The nurse must perform a catheterization on a male client. Which zone of proximity should the nurse use for this intervention? 1. Personal distance 2. Intimate distance 3. Social distance 4. Public distance

Answer: 2

The nurse needs to evaluate the effectiveness of a teaching session with a client. Which approach would provide the best feedback? 1. Client communication 2. Process recording 3. Therapeutic communication 4. Verbal communication

Answer: 2

The nurse is studying the taxonomy of nursing diagnosis. Which additional classifications are associated with the diagnoses that contribute to standardized nursing language? 1. Care maps 2. NIC and NOC 3. Minimum data sets 4. Standardized care plans

Answer: 2 Explanation: Nursing diagnosis is part of a larger, developing system of standardized nursing language that includes classifications of nursing interventions (NIC) and nursing outcomes (NOC). Care maps, minimum data sets, and standardized care plans are not a part of a system of standardized nursing language.

According to the care plan, a client is to receive chest physiotherapy twice daily. The client lives alone in a rural area, does not drive, and is 40 miles away from a hospital. What should the home care nurse do when setting priorities for this client? 1. Ensure for reliable transportation to get to the client's home. 2. Assist the client in finding an alternative plan for achieving the therapy's outcomes. 3. Tell the client that this therapy will be impossible to receive. 4. Make arrangements to have the client moved to a long-term care facility.

Answer: 2 Explanation: The nurse must consider a variety of factors when assigning priorities, including resources available to the nurse and client. Factors in this case include the distance between the client's home and the hospital and the fact that therapy is ordered on a twice-daily basis. Driving 80 miles two times a day may not be feasible, but perhaps there are other alternatives that could be considered (e.g., a neighbor who might be willing to drive the client or someone in the area who may be able to assist with the therapy).

After communicating with the client and family, the nurse compares a client's problem list with identified nursing diagnoses. What action is the nurse performing to minimize diagnostic errors? 1. Understanding what is normal versus what is not normal 2. Verifying 3. Consulting resources 4. Basing diagnoses on patterns

Answer: 2 Explanation: The nurse, while taking the information and collecting data, begins to hypothesize possible explanations of the data and then realizes all diagnoses are only tentative until they are verified. The client and family should be included in the beginning and also at the end of the diagnostic process to verify the nurse's diagnoses.

The nurse being oriented to a new position is reviewing the hospital's standards of care, standardized care plans, protocols, policies, and procedures. For which reasons should the nurse realize that these documents are being used by the nursing staff? (Select all that apply.) 1. Making sure all clients have the same types of care 2. Ensuring that minimally accepted standards are met 3. Promoting efficient use of the nurse's time 4. Eliminating care disparities among clients 5. Ensuring medication errors do not occur

Answer: 2, 3

The nursing staff is reviewing standards of care, standardized care plans, protocols, policies, and procedures for a multisystem healthcare facility. Why are these documents important to the nursing staff when providing client care? (Select all that apply.) 1. Make sure all clients have the same type of care 2. Ensure that minimally accepted standards of care are met 3. Promote efficient use of the nurse's time 4. Eliminate care disparities among clients 5. Minimize healthcare costs

Answer: 2, 3

The nurse manager is conducting a survey of personnel to see what the general feeling is before implementing computerized charting in an acute care hospital. What should the nurse select as positive aspects of implementing this type of system? (Select all that apply.) 1. The system is relatively inexpensive to maintain. 2. Bedside terminals eliminate worksheets and note taking. 3. The system links to various sources of client information. 4. The system better protects client privacy. 5. Information is legible. 6. Results, requests, and client information can be sent and received quickly.

Answer: 2, 3, 5, 6

A client grimaces and responds "I'm fine" when asked by the nurse to describe a level of pain. Which communication factor is the client struggling with? 1. Territoriality 2. Environment 3. Congruence 4. Attitude

Answer: 3

After classroom discussion regarding confidentiality policies and laws protecting client records, a student asks why it's permissible for them to review and have access to client records in the clinical area. How should the nursing instructor respond? 1. "Confidentiality and privacy laws don't apply to students." 2. "Most students review so many records and charts that they could not possibly remember details from any one of them." 3. "Records are used in educational settings and for learning purposes, but the student is bound to hold all information in strict confidence." 4. "As long as the clinical instructor is in the area, accessing client records is part of the education process."

Answer: 3

During the assessment of a client's breasts, the nurse finds both breasts rounded, slightly unequal in size, skin smooth and intact, and nipples without discharge. What should the nurse do next? 1. Notify the charge nurse. 2. Notify the physician. 3. Document the findings in the nurse's notes as normal. 4. Document the findings in the nurse's notes as abnormal.

Answer: 3

During the process of administering medications, the nurse checks the name band for the client's name. What should be this nurse's next action? 1. Administer the medication as ordered. 2. Initial the MAR that the medication will be given. 3. Double-check the client's identification using a second method. 4. Educate the client regarding the medication to be given.

Answer: 3

The nurse explains to a client that he will need to have a bowel prep before going to his esophagogastroscopy. On what should the nurse focus to improve communication skills? 1. Pace 2. Intonation 3. Simplicity 4. Clarity

Answer: 3

The nurse identifies that the ordered dose for a medication is twice the amount generally administered. What action should the nurse take? 1. Administer the medication as it was ordered. 2. Check to see if previous shift nurses gave the medication. 3. Collaborate with the prescriber about the order. 4. Administer only the standard dose of the medication.

Answer: 3

The nurse identifies the diagnosis Risk for aspiration, related to neuromuscular dysfunction for a client who experienced a cerebrovascular accident. Which intervention should the nurse identify as including a rationale? 1. Have suction equipment available at all times. 2. Clear secretions from oral/nasal passageways as needed. 3. Keep client in low-Fowler position to prevent reflux. 4. Provide frequent assessment for presence of obstructive material in mouth and throat.

Answer: 3

The nurse is assessing peripheral pulses on a client with suspected peripheral vascular disease. Which finding should the nurse report to the physician immediately? 1. Pulses equal bilaterally 2. Full pulsations 3. Thready pulses 4. Pulses present bilaterally

Answer: 3

The nurse is conducting an admission interview. Which response indicates that the nurse is attentively listening to the client's explanations? 1. "Can you explain what your symptoms are like?" 2. "When was the last time you saw a doctor for this?" 3. "Uh-huh," while nodding the head 4. "I'm sorry, say that again?"

Answer: 3

The nurse is giving a demonstration of new equipment to the rest of the staff. Which level of proxemics should the nurse use? 1. Intimate 2. Personal 3. Social 4. Public

Answer: 3

The nurse is performing a lung assessment on a client with suspected pneumonia. Which finding should the nurse report to the physician immediately? 1. Chest symmetrical 2. Breath sounds equal bilaterally 3. Asymmetrical chest expansion 4. Bilateral symmetric vocal fremitus

Answer: 3

The nurse is preparing a small amount of medication for oral administration. Which nursing action is essential? 1. Draw up the medication in a syringe with a large-gauge needle. 2. Measure the medication at the top of the meniscus. 3. Label the syringe with the medication name, amount, and route. 4. Dilute the medication with water before measuring.

Answer: 3

The nurse is preparing for morning rounds. What should the nurse avoid delegating to unlicensed assistive personnel? 1. Vital signs 2. Filling of water pitchers 3. Skull and face assessment 4. Ambulation of surgical clients

Answer: 3

The nurse is preparing the morning assignments. Which assessment could the nurse delegate to unlicensed assistive personnel? 1. Neurological assessment 2. Musculoskeletal assessment 3. Vital signs assessment 4. Female genital assessment

Answer: 3

The nurse is preparing to perform a health assessment of the abdomen. In which order should the nurse perform the assessment? 1. Auscultate, percuss, palpate, inspect 2. Inspect, auscultate, palpate, percuss 3. Inspect, auscultate, percuss, palpate 4. Palpate, percuss, auscultate, inspect

Answer: 3

The nurse is reviewing medications prescribed for a client. Why is the nurse writing out the name of the drug morphine sulfate instead of using the abbreviation MS? 1. The hospital has placed MS on its list of do-not-use abbreviations. 2. The Joint Commission (TJC) requires that the abbreviation MS not be used. 3. Using the abbreviation MS puts the client at risk of medication error. 4. Computerized charting systems will not accept the abbreviation MS.

Answer: 3

The nurse is to administer 0.75 mL of medication subcutaneously in the upper arm to a 300-pound adult client. The nurse can grasp approximately 2 inches of the client's tissue at the upper arm. Which is the most appropriate for the nurse to use? 1. A tuberculin syringe, #25-#27 gauge, 1/4- to 5/8-inch needle 2. Two 3-mL syringes, #20-#23 gauge, 1 1/2-inch needle 3. 3.2-mL syringe, #25 gauge, 5/8-inch needle 4. 4.2-mL syringe, #20-#23 gauge, 1-inch needle

Answer: 3

The nurse who is to administer 2.5 mL of intramuscular pain medication to an adult client notes that the previous injection was administered in the right ventrogluteal site. In which site should the nurse plan to administer this injection? 1. The same site 2. The deltoid 3. The left ventrogluteal 4. The rectus femoris

Answer: 3

The nurse manager observes a new graduate nurse complete assessment activities for a newly admitted client. Which actions indicate that the graduate needs assistance with the assessment process? (Select all that apply.) 1. Reviews client record 2. Establishes a database 3. Performs nursing actions 4. Reviews nursing literature 5. Determines client's strengths, risks, and problems

Answer: 3, 5

After formulating several diagnoses, the nurse does not understand the reason for some of the discrepancies in the client's laboratory values and diagnostic tests, when comparing to norms and standards. Which action should the nurse take? 1. Verify the information with the client. 2. Compare all findings to the national norms and standards. 3. Consult other professionals and colleagues. 4. Improve critical thinking skills so answers come more easily.

Answer: 3 Explanation: Both novices and experienced nurses should consult appropriate resources whenever in doubt about a diagnosis. Professional literature, nursing colleagues, and other professionals are all appropriate resources.

The nurse has formulated the diagnosis of Activity intolerance, related to weakness and debilitation, manifested by reports of fatigue after any physical activity. What is the defining characteristic of this label? 1. Activity intolerance 2. Weakness and debilitation 3. Reports of fatigue 4. Physical activity

Answer: 3 Explanation: The defining characteristics are those reports given by the client, or the signs and symptoms. Activity intolerance, weakness and debilitation, and physical activity are not defining characteristics.

The nurse is caring for a client recovering from a long and difficult childbirth experience. Which nursing diagnosis did the nurse write appropriately for this client? 1. Constipation, due to tissue trauma, manifested by no bowel movement for 2 days 2. Risk for infection, because of new incision, related to episiotomy 3. Ineffective breastfeeding, related to lack of motivation, secondary to exhaustion 4. Altered urinary elimination, secondary to childbirth

Answer: 3 Explanation: The problem statement is listed first followed by the etiology-factors that contribute to or are the cause of the client's response. The two parts are joined by the words "related to," implying a relationship between the two. Adding a second part to the etiology statement makes it more descriptive and useful.

The nurse makes chronological entries in a client's chart that include documentation about the routine care provided, assessment findings, and client problems during a 12-hour shift. Which type of charting is this nurse completing? 1. Problem-oriented recording 2. Source-oriented recording 3. Narrative charting 4. Plan of care

Answer: 3 Rationale: Narrative charting is a traditional part of the source-oriented record. It consists of written notes that include routine care, normal findings, and client problems. There is no right or wrong order to the information, although chronological order is frequently used.

The nurse is completing a health history with a newly admitted client. What information should the nurse include when asking about the history of the client's present illness? (Select all that apply.) 1. Allergies 2. Immunization record 3. When the symptoms started 4. Exact location of the problem 5. Things that aggravate the problem

Answer: 3, 4, 5

The nurse formulates nursing diagnoses for a client with chronic renal failure. Which statements indicate the nurse appropriately used a two-part format? (Select all that apply.) 1. Pruritis related to toxin buildup in the blood 2. Hypertension related to fluid volume overload 3. Deficient fluid volume related to fluid restriction 4. Personal care challenges related to fistula in left arm 5. Acute confusion related to delayed hemodialysis treatment

Answer: 3, 5

A nursing student is preparing to administer insulin to a client with diabetes. Indicate the correct order for the administration of this medication: 1. Cleanse the site with alcohol. 2. Insert the needle quickly into the subcutaneous tissue. 3. Mix the insulins. 4. Assess the skin for the injection. 5. Pinch the skin lightly. 6. Inject the medication. 7. Count to five. 8. Remove the syringe. Place in the correct order.

Answer: 3,4,1,5,2,6,7

The graduate nurse is thinking about leaving a new job because of actions demonstrated by the nurse manager. Which actions should the graduate nurse identify as bullying? (Select all that apply.) 1. Pairing the graduate with a seasoned nurse to assist with learning new skills 2. Asking the graduate to participate in client rounds with the new interns on the care area 3. Confronting the graduate by stating that refusing an assignment is grounds for dismissal 4. Stating that requests for vacation time will be denied because the nurse asks too many questions 5. Assigning the graduate nurse a complicated client with needs that the graduate is not comfortable performing

Answer: 3,4,5

A client has just lost a second baby to preterm complications. Which statement demonstrates the best therapeutic response for the nurse to make? 1. "Don't be so sad. You can always try again." 2. "Didn't your doctor advise you about genetic counseling?" 3. "I know how you feel. I have children of my own." 4. "I am so sad for you. I'll stay with you for a while if you need to talk."

Answer: 4

A client recovering from a stroke is able to comprehend what is being said but is unable to respond by speech or writing. What type of aphasia should the nurse realize this patient is demonstrating? 1. Auditory aphasia 2. Acoustic aphasia 3. Sensory aphasia 4. Expressive aphasia

Answer: 4

A teenage client has been having problems with peer support, school performance, and parental expectations, all of which contributed to an eating disorder. After gathering this assessment data, the nurse formulates the diagnosis Activity Intolerance related to weakness. What should the nurse realize after evaluating this diagnosis? 1. The data collected would support the diagnosis. 2. The diagnosis is directly related to the data presented. 3. The nursing diagnosis is not relevant to the data. 4. The data are not sufficient enough to support this diagnosis.

Answer: 4

After completing the client care and documenting it in the progress notes, the nurse realizes that documentation was placed on the wrong medical record. What should the nurse do? 1. Use white-out over the mistake. 2. Take a wide permanent marker and blacken out all the documentation. 3. Put an "X" through the entire page, identify it as an "error," initial, and move on to the correct chart. 4. Draw a single line through the documentation, write "mistaken entry" next to the original entry, and initial it.

Answer: 4

During teaching, the nurse makes sure the client understands how to activate the safety mechanism on the syringe to prevent needle stick injuries when self-administering insulin. Which guideline of implementing interventions is the nurse using? 1. Adapt activities to the individual client. 2. Encourage clients to participate actively in implementing nursing interventions. 3. Base nursing interventions on scientific knowledge, research, and standards of care. 4. Implement safe care.

Answer: 4

The following medications are listed on a client's medication administration record (MAR). Which medication order should the nurse question? 1. Lasix 40 mg, po, STAT 2. Ampicillin 500 mg, q6h, IVPB 3. Humulin L (Lente) insulin 36 units, subcutaneously, every morning before breakfast 4. Codeine q4-6h, po, prn for pain

Answer: 4

The nurse enters a client's room and finds that the telephone is lying in the client's lap, tissues are wadded up on the bed, and the client's eyes are red and watery. What is the best response by the nurse? 1. "Can I hang that phone up for you?" 2. "Well, it's a beautiful day outside. Let's open the blinds." 3. "Has your doctor been in to talk to you yet?" 4. "You look upset. Is there anything you'd like to talk about?"

Answer: 4

The nurse is caring for a client who has just received a new cardiac medication. How should the nurse instruct the unlicensed assistive personnel (UAP) who is also caring for this client? 1. Have the UAP assess for any unexpected effects from the medication. 2. Tell the UAP to teach the client's family what to expect from the medication. 3. Have the UAP look the medication up in a drug reference book to read about drug actions and possible side effects. 4. Give the UAP specific instructions regarding what drug actions or side effects to report to the nurse.

Answer: 4

The nurse needs to communicate information about a client's status to a physician. Which approach demonstrates assertive communication by the nurse? 1. "You need to check the laboratory results of the client in room 423." 2. "You should visit with the client's family about the upcoming procedure." 3. "We need to be more aware of the situation among the client and the client's family." 4. "I am concerned that the client does not have adequate pain management."

Answer: 4

While administering an intramuscular injection, the nurse notes blood return in the syringe barrel after aspirating. What action should the nurse take? 1. Pull the needle out inch and inject the medication. 2. Inject the medication as planned. 3. Notify the physician immediately. 4. Discard the medication and start over.

Answer: 4

While preparing to administer a transdermal estrogen patch, the nurse finds the previously applied patch in the client's bed linens. How can the nurse avoid this situation with the patch now being applied? 1. Shave the area where the patch is being applied. 2. Place a heating pad over the area where the patch is applied for 10 minutes after application. 3. Run a finger around the adhesive edges of the new patch before placing it on the client's skin. 4. Press firmly over the patch with the palm of the hand for about 10 seconds after applying it to the skin.

Answer: 4

The nurse identifies for a client the nursing diagnosis "Fluid volume deficit, related to active fluid loss, secondary to diarrhea." What would be an appropriate goal statement for this diagnosis? 1. Client will drink more fluids by tomorrow. 2. Client will have good skin turgor. 3. Client will have moist mucous membranes. 4. Client will have intake of at least 1000 mL within 24 hours.

Answer: 4 Explanation: The goal statement must be specific with observable outcomes in order for the nurse to evaluate client progress, and all options must have a time frame for evaluating the desired performance. This option includes all necessary components.

During a health history, a client admits to taking nutritional supplements instead of prescribed medication. Which responses by the nurse indicate effective communication? (Select all that apply.) 1. "What you did was wrong." 2. "Who do you think you are?" 3. "You shouldn't have done that." 4. "Tell me more about the supplements." 5. "Explain the reasoning behind your decision."

Answer: 4, 5

The nurse is performing a health assessment and notes a yellow tinge to the sclera of the eye. In which way should the nurse document this finding? 1. Cyanosis 2. Jaundice 3. Pallor 4. Erythema

Answer:2

While performing an assessment of the integument system, the nurse notes the client's eyeballs are protruding and the upper eyelids are elevated. What term should the nurse use to document this finding? 1. Erythema 2. Cyanosis 3. Exophthalmos 4. Normocephalic

Answer:3

STAT order

Any medication that is needed immediately and is to be given only once.

S4 heart sound

Atrial Gallup; related to stiffness of the ventricular myocardium to rapid filling

The nurse questions if the dosage of a medication is unsafe for the client because of the client's weight and age. The nurse should take which of the following actions? A. Administer the medication as ordered by the prescriber B. Call the prescriber to discuss the order and the nurse's concern C. Administer the medication, but chart the nurse's concern about the dosage D. Give the client half the dosage and document accordingly

B. Call the prescriber to discuss the order and the nurse's concern Rationale: Client safety is of the utmost importance when implementing any nursing intervention. If the nurse feels that an order is unsafe or inappropriate for a client, the nurse must act as a client advocate and collaborate with the appropriate healthcare team member to determine the rationale for the order and/or modify the order as necessary. A nurse accepts accountability for his or her actions. Options 1, 3, and 4 are inappropriate and unsafe.

The nurse would place which correctly written nursing diagnostic statement into the client's care plan? A. Cancer relater to cigarette smoking B. Impaired gas exchange related to aspiration of foreign matter as evidenced by oxygen saturation of 91% C. Imbalance nutrition: more than body requirement related to overweight status D. Impaired physical mobility related to generalized weakness and pain

B. Impaired gas exchange related to aspiration of foreign matter as evidence by oxygen saturation of 91% Rationale: A nursing diagnosis consists of two parts joined by related to. The first part (the human response) names/labels the problem. The second part (related factors) includes the factors that either contribute to or are probable etiologies of the human response. Some formats include a third part to the statement for actual (not risk) diagnoses; this third part consists of the client's signs or symptoms and is joined to the statement with the label as evidenced by. This type of statement is the most complete. Option 1 is not a nursing diagnosis but is a medical diagnosis. Options 3 and 4 are vague.

The nurse who documents on the client's care plan the outcome goal "Anxiety will be relieved within 20 to 40 minutes following administration of lorazepam (Ativan)" is engaged in which step of the nursing process? A. Assessment B. Planning C. Implementation D. Evaluation

B. Planning Rationale: The planning step of the nursing process involves formulating client goals and designing the nursing interventions required to prevent, reduce, or eliminate the client's health problems. Outcome goals are documented on the client's care plan. Assessment data (option 1) is used to help identify a client's human response, and once a plan is established, the interventions are implemented (option 3) and evaluated (option 4).

Semi-Fowler's Position

Back-lying with head of bed elevated around 30-45 degrees Area Assessed: Jugular vein distension

When responding to a call light, the nurse finds a client lying on the floor, with the bed linens around the legs. Which chart entry should the nurse document for this finding? 1. Client fell out of bed but did push the call button for assistance. 2. Client became tangled in the bed linens, then called for assistance after falling out of bed. 3. Recorder responded to client's call light, upon entering the room, found client on floor. 4. Client found on floor, appeared to have fallen out of bed as a result of getting tangled in bed linens.

Correct Answer: 3 Rationale 1: It should never be assumed that the client fell out of bed. Rationale 2: It should never be assumed that the client fell out of bed, became tangled in bedding, or anything else. Rationale 3: Accurate notations consist of facts or observations rather than opinions or interpretations. The client was found on the floor, and the call light was activated. Those are the only things known until the nurse learns further information from questioning the client. Rationale 4: It should never be assumed that the client became tangled in bedding, or anything else.

6 cardinal fields of gaze

stand in front of pt and have them follow the object; make sure you tell them to follow only with their eyes. patterns include the H-pattern or the wheel pattern; you pause at each area so you can watch their eyes at each of these positions

Head to toe assessment : abdomen

Inspect; auscultate; percussion; palpate

Sequence of Head to Toe

Inspect; palpate ; percussion; auscultation

Zone of proximity: Intimate

Interaction between parents and children or people who desire close personal contact or touching

At what age should men screen for prostate cancer?

Men aged 50 + who have at least a 10 year life expectancy, should speak with provider about benefits of screening

Half-life of a medication

the amount of time it takes for half of the drug to be eliminated. Liver and kidney disease, aging, absence of food, and slowed metabolic rate prolong half-life because of their effects on metabolism and excretion.

Tongue blades

to depress the tongue during assessment of the mouth and pharynx

How is distribution of the oral medication affected if a client has less than normal cardiac output (low BP, prolonged capillary refill)?

The medication will not be circulated through the body effectively.

Cranial Nerve X

Vagus - senses aortic blood pressure & slows heart rate & stimulates digestive organs & taste

syndrome diagnosis

a diagnosis that is associated with a cluster of other diagnoses

Strabismus

abnormal deviation of the eye

Cranial Nerve XI

accessory nerve

General Survey: Physical Appearance

age, sex, level of consciousness, skin color, facial features

concurrent audit

evaluation of a client's health care while the client is still receiving care from the agency

Nursing Procedures

guided by evidence based literature, designed to provide safe, high quality care, frequently evaluated for best practices

Otoscope

instrument used for visual examination of the ear

Formal nursing care plan

is a written or computerized guide that organizes information about the client's care.

Pharmacopoeia

medical reference summarizing standards of drug purity, strength, and directions for synthesis

Ulcer

open sore or lesion in the skin or mucous membrane


Kaugnay na mga set ng pag-aaral

PrepU Chp 28: Assessment of Hematologic Function and Treatment Modalities

View Set

Interpersonal Communication/ Chapter 9

View Set

A&P Respiratory and Digestive System

View Set

Class IV Antidysrhythmic Drugs (Calcium Channel Blockers)

View Set